筛选结果 共找出33

Edward sells two products with selling prices and contributions as follows:  



Edwards’s fixed costs are $1,400,000 per year.

 Edward now anticipates that more customers will buy the cheaper product G and that budgeted sales will be 150,000 units for each product. 

If this happens what would happen to the breakeven revenue? 

A

 Increase by the extra revenue from G of 50,000 × $20/u or $1,000,000 

B

 Decrease by the extra revenue from G of 50,000 × $20/u or $1,000,000 

C

 Increase by a different amount 

D

 Decrease by a different amount 

The following breakeven chart has been drawn for a company’s single product: 



Which of the following statements about the product are correct?

 (i) The product’s selling price is $10 per unit. 

(ii) The product’s variable cost is $8 per unit. 

(iii) The product incurs fixed costs of $30,000 per period. 

(iv) The product earns a profit of $70,000 at a level of activity of 10,000 units. 

A

 (i), (ii) and (iii) only 

B

 (i) and (iii) only 

C

 (i), (iii) and (iv) only 

D

 (i), (ii) and (iv) only 

 C/S ratio = P/V ratio × 100

A

True

B

 false

Which of the following is not a major assumption of breakeven analysis? 

A

 It can only apply to one product or a constant sales mix. 

B

 Fixed costs are the same in total and unit variable costs are the same at all levels of output. 

C

 Sales prices vary in line with levels of activity. 

D

 Production level is equal to sales level. 

 HG plc manufactures four products. The unit cost, selling price and bottleneck resource details per unit are as follows.  



Assuming that labour is a unit variable cost, if budgeted unit sales are in the ratio W : 2, X : 3, Y : 3, Z : 4 and monthly fixed costs are budgeted to be $15,000, the number of units of W that would be sold per month at the budgeted breakeven point is nearest to: 

A

 106 units 

B

 142 units 

C

 212 units 

D

 283 units 

 Co X makes two products Y and Z, which it sells in the ratio 4:2. (This ratio is based on the sales revenue.) The sales prices and variables costs of Y and Z are as follows:  

                                               Sales price                               Variable costs

 Y                                                      $61                                             $42 

Z                                                       $95                                             $63 

Fixed costs for the business are $200,000. 

What is the breakeven revenue for the business (to the nearest whole number)? 

A

 $322,000 

B

$612,000 

C

 $620,000 

D

 $857,000 

This question appeared in the June 2015 exam. 

The following information is available for a manufacturing company which produces multiple products: 

(1) The product mix ratio 

(2) Contribution to sales ratio for each product 

(3) General fixed costs 

(4) Method of reapportioning general fixed costs 

Which of the above are required in order to calculate the breakeven sales revenue for the company? 

A

 All of the above 

B

1, 2 and 3 only 

C

 1, 3 and 4 only 

D

 2 and 3 only 

 An organisation manufactures and sells a single product, the G. It has produced the following budget for the coming year: 



If inventory levels are negligible, what is the breakeven point in units? 

A

 13,634 

B

 13,750 

C

 17,500 

D

 28,000 

 A company manufactures and sells a single product with a variable cost per unit of $36. It has a contribution ratio of 25%. The company has weekly fixed costs of $18,000.  

What is the weekly breakeven point, in units? 

A

 1,500 

B

 1,600 

C

 1,800 

D

 2,000 

A company makes a single product with the following data:  

                                                                                      $                           $ 

Selling price                                                                                           25 

Material                                                                        5 

Labour                                                                         7 

Variable overhead                                                     3 

Fixed overhead                                                          4  

                                                                                                                 (19) 

                                                                                                                 ––– 

Profit per unit                                                                                           6  

                                                                                                                 ––– 

Budgeted output is 30,000 units. 

In relation to this data, which of the following statements is correct? 

A

 The margin of safety is 40% 

B

 The contribution to sales ratio is 24% 

C

 The volume of sales needed to make a profit of $270,000 is 45,000 units 

D

 If budgeted sales increase to 40,000 units, budgeted profit will increase by $100,000